5. What is the net force on an object with a friction of 50 N toward the left and an applied force of 80 N towards the right?130 N right130 N left30 N left30 N right

Answers

Answer 1

Given,

The frictional force is 50 N towards left.

The applied force is 80 N towards right.

Thus the net force is:

F=80N-50N=30 N toards right.

The answer is 30 N towards right.


Related Questions

You're standing on a ramp that is inclined at a 25 degree angle. The weight (Fg) experienced is 50N. What is the parallel force (Fx) and the perpendicular force (Fy)?I need to do the following: 1. Draw a free body diagram2.Identify Givens and Unknowns3.Identify the Equations4.Set up the equation using the givens and unknowns5.Solve

Answers

Answer:

Fx = 21.13 N

Fy = 45.32 N

Explanation:

In this case, the free body diagram is:

We can see that there is a right triangle formed by the weight (Fg) and the ramp. So, we can calculate the value of angle θ.

25 + 90 + θ = 180

115 + θ = 180

θ = 180 - 115

θ = 65

Because the sum of the interior angles of a triangle is always 180 degrees.

Now, we can calculate the parallel force and the perpendicular force using the trigonometric functions sine and cosine, so

Fx = Fg cos θ

Fy = Fg sin θ

So, replacing Fg = 50 N and θ = 65, we get:

Fx = 50 cos(65) = 21.13 N

Fy = 50 sin(65) = 45.32 N

Therefore, the answers are:

Fx = 21.13 N

Fy = 45.32 N

Two figure skaters, one weighing 625 N and the other 725 N, push off against each other onfrictionless ice. If the heavier skater travels at 1.5 m/s, how fast will the lighter one travel?A 1.7 m/sB 2.8 m/sC -1.7 m/sD -2.8 m/s

Answers

We are given the following information:

Weight of skater 1 = 625 N

Weight of skater 2 = 725 N

Final velocity of skater 2 = 1.5 m/s

Final velocity of skater 1 = ?

Recall from the law of conservation of momentum, the total momentum before the collision and after the collision must be equal.

[tex]\begin{gathered} p_{before}=p_{after} \\ m_1u_1+m_2u_2=m_1v_1+m_2v_2 \end{gathered}[/tex]

The initial velocities of both skaters are 0 m/s

[tex]m_1\cdot0_{}+m_2\cdot0=m_1v_1+m_2v_2[/tex]

Also, m = W/g

[tex]\begin{gathered} 0=m_1v_1+m_2v_2 \\ 0=(\frac{625}{9.8})\cdot_{}v_1+(\frac{725}{9.8})\cdot1.5 \\ (\frac{625}{9.8})\cdot_{}v_1=-(\frac{725}{9.8})\cdot1.5 \\ (63.78)\cdot_{}v_1=-110.97 \\ _{}v_1=-\frac{110.97}{63.78} \\ _{}v_1=-1.7\: \frac{m}{s} \end{gathered}[/tex]

So, the lighter skater will travel with a velocity of 1.7 m/s

The negative sign means that the lighter skater will be traveling oppositely to the heavier skater.

in four hours, a hiker in a canyon goes from 892ft to 256 ft above the canyon floor. Find the hikers vertical speed.

Answers

From the given question, we can deduce the following information:

• Time, t = 4 hours

,

• The hiker goes from 892 ft to 256 ft above the canyon floor.

Let's find the vertical speed of the hiker.

To find the speed, apply the formula:

[tex]speed=\frac{dis\tan ce}{time}[/tex]

To find the distance, we have:

Distance covered = 256 ft - 892 ft = -636 ft.

Hence, to find the vertical speed, we have:

[tex]\begin{gathered} speed=\frac{dis\tan ce}{time} \\ \\ \text{speed}=\frac{-636}{4}=-159\text{ ft/h} \\ \end{gathered}[/tex]

Therefore, the vertical speed of the hiker is -159 feet per hour.

ANSWER:

What happens to the strength of an electromagnet if the current in the wire is increased?

Answers

We will have the following:

What will happen is that the magnetic field will increase; since the magnetic field increases when the current in the wire of the electromagnet increases.

how do I convert fractions into percentages like 14 over 20

Answers

Given data

*The given fraction is

[tex]x=\frac{14}{20}[/tex]

Convert the given fraction into percentage as

[tex]\begin{gathered} x=\frac{14}{20}\times100\text{\%} \\ =0.7\times100\text{\%} \\ =70\text{\%} \end{gathered}[/tex]

which choice are equivalent to the expression below? check all that apply. 4 square root 5answer choices: square root of 16*6, square root of 32*3, square root of 96, square of 24, 96, square root of 4*36

Answers

Given

4 square root 6

[tex]4\sqrt[]{6}[/tex]

Wich choices are equivalent to the expression below

[tex]\begin{gathered} \sqrt[]{16}\sqrt[]{6} \\ \sqrt[]{32}\sqrt[]{3} \\ \sqrt[]{96} \end{gathered}[/tex]

The first three options are the correct answers.

Find the y-component of thisvector:12.0 m73.3°Remember, angles are measured fromthe +X axis.y-component (m)

Answers

Given data:

The magnitude of the given vector is,

[tex]A=12.0\text{ m}[/tex]

An angle between the given vector and +x axis is,

[tex]\theta=73.3^o[/tex]

The formula of y-component is as follows:

[tex]A_y=A\sin \theta[/tex]

Here,

[tex]A\text{ is the magnitude of the given vector-A}[/tex]

Now, substitute known values in above equation;

[tex]\begin{gathered} A_y=12\text{ m}\times\sin 73.3^o \\ A_y=11.49\text{ m} \end{gathered}[/tex]

Therefore, the y-component of the given vector is 11.49 m

URGENT!! ILL GIVE
BRAINLIEST!!!! AND 100 POINTS!!!!!!

Answers

Answer:

b heated gas will have decreased kinetic energy and decreased density

Explanation:

gas loses weight

One way to create more fissionable fuel in the form of plutonium-239 is in which of the following?Select one:a.a coal-fire power station b.radioactive wastec.a meltdown of the cored.a breeder reactor

Answers

The correct answer is radioactive waste.

The Plutonium-239 is waste material of fuel road used in a nuclear power plant for the electricity production. The use of nuclear power plant will increase the radioactive waste and the production of Plutonum-239 will be more.

Thus, option b is

John is at the park playing fetch with his cat. He throws the ball 1m south, and his cat retrieves the ball and returns it to John. John then throws the ball 3 meters, and the cat again collects the ball and returns it to John. When the cat returns for the second time, what distance has the cat traveled?6m8m2m4m

Answers

As John throws the ball 3 meters and the cat collects the ball and returns to John. Then. the distance traveled by the cat when it returns from the second time is calculated as

[tex]\begin{gathered} d=3+3 \\ =6\text{ m} \end{gathered}[/tex]

How much does James have to pay for a vaccum cleaner that he uses for 4 hours a week.  In James's neighborhood, the cost of electricity is 27 cents per kwhr.The power is P = 800 W

Answers

Given:

The power of the vacuum cleaner is P = 800 W = 0.8 kW

The time consumption is t = 4 hours a week.

The cost of electricity is 27 cents per kWh

To find the cost of the electricity for using the vacuum cleaner for a week.

Explanation:

First, we need to find the energy.

The energy can be calculated as

[tex]\begin{gathered} E=P\times t \\ =0.8\times4 \\ =3.2\text{ kWh} \end{gathered}[/tex]

The cost of electricity per week due to vacuum cleaner will be

[tex]27\times3.2=86.4\text{ cents}[/tex]

Final Answer: The cost of electricity James has to pay for a vacuum cleaner for using it 4 hours a week is 86.4 cents.

General relativity is combines special relativity with the equivalence principle.Why and why not?

Answers

The equivalence principle is a fundamental law of physics that states that inertial forces and gravitational forces are similar in nature and are often indistinguishable from each other.

The application of the equivalence principle to in combination with the general theory of relativity allowed the refinement of the equations of the theory helping give birth to the theory of general relativity.

For example, in special relativity objects are considered to be moving with a constant velocity. When the principle of equivalence is introduced this allows considering inertial frames or accelerated frames of reference and the gravitational forces in a system thus completing the special theory of relativity.

When the acceleration in the general theory of relativity is considered to be zero then it reduces to the special theory of relativity.

Calculate the torque experienced by the door due to this force using torque is equal to force times lever armLength of the lever arm = 1 mForce = 5 N

Answers

Given:

The applied force on the door is F = 5 N

The length of the lever arm is l = 1 m

Required: Torque experienced by the torque.

Explanation:

Torque is the product of force and the distance between the force applied and the rotational axis.

The force is applied on one side of the lever arm while the rotational axis will be at the other end of the lever arm.

So, the distance between the rotational axis and the force applied is the length of the arm.

Torque can be calculated by the formula

[tex]\tau=F\times l[/tex]

On substituting the values, the torque will be

[tex]\begin{gathered} \tau=5\times1 \\ =5\text{ N m } \end{gathered}[/tex]

Final Answer: The torque experienced by the door is 5 N m.

Pascal's principle states that:all liquids exert pressure downward.pressure will move toward areas where pressure is lowest.applied pressure will be transmitted throughout a fluid.None of the choices are correct.

Answers

According to Pascal's principle states that applied pressure will be transmitted throughout a fluid.

Thus, third one is the correct option.

What are electromagnetic radiations?​

Answers

Radiations associated with electric and magnetic field is called Electromagnetic radiations.

electric and magnetic field radiations = electromagnetic radiations.

The fundamental frequency of a pipe is the lowest resonant frequency. Is this true or false?

Answers

By definition, the fundamental frequency of an object is the lowest frequency such that the object resonates.

Then, the answer is:

[tex]\text{True}[/tex]

Answer:

By definition, the fundamental frequency of an object is the lowest frequency such that the object resonates.

Then, the answer is: true

Explanation:

I have some problems applying the formulas to solve physics problems. I understand all the concepts needed, but just freeze when I see questions, especially when it comes to trying to combine linear and rotational conceptsA uniform, 255 N rod that is 1.90 m long carries a 225 N weight at its right end and an unknown weight W toward the left end (Figure 1). When W is placed 60.0 cm from the left end of the rod, the system just balances horizontally when the fulcrum is located 75.0 cm from the right end.1) Find W.2) If W is now moved 30.0 cm to the right, how far must the fulcrum be moved to restore balance?

Answers

ANSWER

[tex]\begin{gathered} 1)\text{ }214.90\text{ }N \\ \\ 2)\text{ }0.09\text{ }m \end{gathered}[/tex]

EXPLANATION

First, let us make a sketch of the diagram showing the distances on the rod:

1) Since the fulcrum is balanced, the center of gravity of the system will be at the fulcrum.

The center of gravity (in the horizontal is given by:

[tex]x=\frac{W_1x_1+W_2x_2+W_3x_3}{W_1+W_2+W_3}[/tex]

where W1 = the weight on the right end = 225 N

W2 = the weight of the rod = 255 N

W3 = the weight place on the left = W

x1 = the position of W1 (taking the left as the origin) = 1.90 m

x2 = the position of the center of mass of the rod = x1/2 = 0.95 m

x3 = the position of W from the left end = 0.60 m

x = position of center of gravity of the rod from the left end i.e. at the fulcrum = 1.90 - 0.75 = 1.15 m

Now, substitute the values given in the question and solve for W:

[tex]\begin{gathered} 1.15=\frac{(225*1.90)+(255*0.95)+(W*0.60)}{225+255+W} \\ \\ 1.15=\frac{427.5+242.25+0.60W}{480+W} \\ \\ 1.15(480+W)=669.75+0.60W \\ \\ 552+1.15W=669.75+0.60W \\ \\ 1.15W-0.60W=669.75-552 \\ \\ 0.55W=117.75 \\ \\ W=\frac{117.75}{0.55} \\ \\ W=214.09\text{ }N \end{gathered}[/tex]

That is the value of W.

2) Now, W is moved 30.0 cm (0.30 m) to the right.

This implies that:

[tex]x_3=0.60+0.30=0.90\text{ }m[/tex]

Since the other values (including W) do not change, we can now solve for x, which is the new center of gravity:

[tex]\begin{gathered} x=\frac{(225\times1.90)+(255\times0.95)+(214.09\times0.90)}{225+255+214.09} \\ \\ x=\frac{427.5+242.25+192.681}{694.09}=\frac{862.431}{694.09} \\ \\ x=1.24\text{ }m \end{gathered}[/tex]

Therefore, the fulcrum must be moved:

[tex]\begin{gathered} 1.24\text{ }m-1.15\text{ }m \\ \\ 0.09\text{ }m \end{gathered}[/tex]

The fulcrum should be moved 0.09 m to the right (since the W is moved to the right).

What is the maximum speed at which a cyclist can move on a bend and at an angle from the vertical, he should deviate to the side of the bend, so as not to fall, if the coefficient of friction of the wheel from the road is 0.4 and the radius of curvature of the road is 100m

Answers

Given,

The coefficient of friction between the wheel and the road, μ=0.4

The radius of curvature of the road, r=100 m

The centripetal force for the cyclist to move in the curved path is provided by the friction between the road and the wheel.

Thus,

[tex]\begin{gathered} \frac{mv^2}{r}=\mu mg \\ \Rightarrow v^2=r\mu g \\ v=\sqrt[]{r\mu g} \end{gathered}[/tex]

Where m is the mass of the cyclist and the cycle and g is the acceleration due to gravity.

i need help please. i tried different answers but i cant get it.

Answers

Let's make a diagram to visualize the angle.

As you can observe, the angle is closer to the South orientation.

Therefore, the new angle is 42 degrees from South.

The mass of a satellite orbiting Earth is 15000 kg.

Answers

Newton's universal law of gravity

[tex]F=G\frac{m1\cdot m2}{r^2^{}}[/tex]

Where:

F= force between objects

m1= mass 1 = 15,000 kg

m2= mass 2 = 6x10^24

r = distance = 34,000,000 m

G= universal contant of gravitation = 6.67 x10^-11

Replacing:

[tex]F=6.67\cdot10^{-11}\cdot\frac{15,000\cdot6x10^{24}}{(34,000,000)^2}[/tex]

F= 5,193 N

you have entered a 134-mile biathlon that consists of a run and a bicycle race. During your run, your average velocity is 6 miles per hour, and during your bicycle ace, your average velocity is 29 miles per hour. You finish the race in 7 hours. What is the distance of the run? What is the distance of the bicycle race?upposeWhat is the distance of the run?miles

Answers

ANSWER:

Distance of the run: 18 miles

Distance of the bicycle race: 116 miles

STEP-BY-STEP EXPLANATION:

Given:

Total distance = 134 miles

Total time = 7 hours

Average velocity during running = 6 mph

Average velocity during bicycle = 29 mph

Let x be the running distance and y be the bicycle distance.

We know that velocity equals distance in a given time, like this:

[tex]\begin{gathered} v=\frac{d}{t} \\ \\ \text{ Therefore:} \\ \\ t=\frac{d}{v} \end{gathered}[/tex]

Knowing the above, we can establish the following system of equations:

[tex]\begin{gathered} t_1+t_2=7\rightarrow\frac{d_1}{v_1}+\frac{d_2}{v_2}=7\rightarrow\frac{x}{6}+\frac{y}{29}=7\text{ \lparen1\rparen} \\ \\ x+y=134\rightarrow x=134-y\text{ \lparen2\rparen} \end{gathered}[/tex]

We substitute the second equation in the first and obtain the following:

[tex]\begin{gathered} \frac{134-y}{6}+\frac{y}{29}=7 \\ \\ \frac{(134-y)(29)+6y}{6\cdot29}=7 \\ \\ \frac{3886-29y+6y}{174}=7 \\ \\ 3886-23y=7\cdot174 \\ \\ y=\frac{1218-3886}{-23}=\frac{-2668}{-23} \\ \\ y=116\rightarrow\text{ bicycle distance} \\ \\ \text{ now, for x:} \\ \\ x=134-116 \\ \\ x=18\rightarrow\text{ running distance} \end{gathered}[/tex]

Therefore:

The distance of running is 18 miles and the distance by bicycle is 116 miles.

2. A Carnot engine operates between a 1500 C reservoir and a -20 C reservoir. What is the efficiency of the engine?1. 83%2. 86 %3. 99 %4. 1.3 %

Answers

The efficiency of a Carnot engine is given by

[tex]\eta=1-\frac{T_C}{T_H}[/tex]

Where TC is the temperature of the cold source in kelvin and TH is the temperature of the hot source in kelvin.

Let us first convert the given temperatures from celsius to kelvin.

[tex]\begin{gathered} T_C=-20+273=253K \\ T_H=1500+273=1773K \end{gathered}[/tex]

So, the efficiency is

[tex]\begin{gathered} \eta=1-\frac{253}{1773} \\ \eta=0.857 \\ \eta=85.7\% \end{gathered}[/tex]

Rounding off to a whole number

[tex]\eta=86\%[/tex]

Therefore, the efficiency of the engine is 86%

That is correct. Thank you

The solar system formed about 4.5 billion years ago. This expressed asA. 4.5x10^6 yearsB. 4.5x10^7 yearsC. 4.5x10^8 yearsD. 4.5 x10^9 years

Answers

Given,

The age of the solar system is 4.5 billion years.

One billion is 1,000,000,000.

In scientific notation, we can write one billion as 1×10⁹.

Therefore to express any number in billions we need to multiply it with the above number.

Therefore, we can write 4.5 billion years as,

[tex]4.5\times1\times10^9=4.5\times10^9\text{ years}[/tex]

Therefore we can express 4.5 billion years as 4.5

According to Coulomb's law, the electrical force between two charged objects:A.is zero if they are opposite charges.B.increases with increasing charge.C.does not depend on the amount of charge.D.increases with increasing distance.

Answers

Answer and explanation:

A correct option is an option (B).

The electrical force between two charges is given as,

[tex]F=\frac{1}{4\pi\epsilon_0}\frac{q_1q_1}{r^2}[/tex]

The electrical force is directly proportional to the product of two charges. Thus Force will depend on two charges irrespective of their signs.

Option (A) is incorrect because if charges are opposite, the value of force will not be zero. It will be -ve.

Option (C) is incorrect because the force is directly proportional to the product of charges, it depends on the amount of charge.

Option (D) is also incorrect because the force in inversly proportional to the distance between two charges. Thus, if the distance between two charges is increased, the force between two charges will decrease.

Concllusion:

The correct option is option (B).

Part C and D please, part A=291.2 and part B=33.8

Answers

Given,

The mass of the skater, m=63 kg

The coefficient of static friction, μs=0.4

The coefficient of the kinetic friction, μk=0.02

F₁=242 N

F₂=162 N

(c) The static friction is given by,

[tex]f_s=N\mu_s[/tex]

Where N is the normal force.

The normal force acting on the skater is

[tex]N=mg[/tex]

Where g is the acceleration due to gravity.

Therefore the static friction is given by,

[tex]f_s=mg\mu_s[/tex]

On substituting the known values,

[tex]\begin{gathered} f_s=63\times9.8\times0.4 \\ =246.96\text{ N} \end{gathered}[/tex]

Therefore the static friction on the skater is 246.96 N

d)The net force acting on the skater is

[tex]\begin{gathered} F_{\text{net}}=ma_{} \\ =F_{\text{tot}}-f \\ =F_{\text{tot}}-N\mu_k \\ =F_{\text{tot}}-mg\mu_k \end{gathered}[/tex]

On substituting the known values,

[tex]\begin{gathered} 63a=291.2-63\times9.8\times0.02 \\ a=\frac{278.85}{63} \\ =4.43m/s^2 \end{gathered}[/tex]

Thus the acceleration of the skater is 4.43 m/s²

What is the equation for finding the gravitational force of an object? (1 point)

Answers

The equation for finding the gravitational force of an object is F = mg.

option D is the correct answer.

What is gravitational force of an object?

Gravitational force is an attractive force that exists between all objects with mass due to the influence of the Earth's gravitational pull.

An object with mass attracts another object with mass.

The magnitude of the force is directly proportional to the masses of the two objects and inversely proportional to the square of the distance between the two objects according to Newton's law of universal gravitation.

Mathematically, the formula for gravitational force is given as;

F = mg

where;

m is the mass of the objectg is acceleration due to gravity

Learn more about gravitational force here: https://brainly.com/question/72250

#SPJ1

Victor is driving south. He is traveling at 12 m/s, when he enters an area with a new speed limit. Aver a period of 6 seconds, his speed increases from 12 m/s to 29 m/s. What is Victor’s acceleration during this period?

Answers

Answer:

1.1417 m/s²

Explanation:

Acceleration = final velocity - initial velocity/ time

a= [tex]\frac{v-u}{t}[/tex]

a= [tex]\frac{29-12}{12}[/tex]

a= [tex]\frac{17}{12}[/tex]

a= 1.417 m/s²   or [tex]\frac{17}{12}[/tex] m/s² in fraction form

An object moving with uniform acceleration has a velocity of 13.0 cm/s in the positive x-direction when its x-coordinate is 2.76 cm. If its x-coordinate 3.05 s later is −5.00 cm, what is its acceleration?______ cm/s2

Answers

Since the object is moving with uniform acceleration we have an uniformly accelerated motion which means that we can use the following equations:

[tex]\begin{gathered} a=\frac{v_f-v_0}{t} \\ x=x_0+v_0t+\frac{1}{2}at^2 \\ v_f^2-v_0^2=2a(x-x_0) \end{gathered}[/tex]

Now, in this case we know:

• The initial position 2.76 cm.

,

• The initial velocity 13 cm/s

,

• The final position -5 cm

,

• The time it takes 3.05 s.

And we want to determine the acceleration; from what we know and what we want we determine that we can use the second equation. Plugging the values in that equation we have that:

[tex]\begin{gathered} -5=2.76+(13)(3.05)+\frac{1}{2}(3.05)^2a \\ \frac{3.05^2}{2}a=-5-2.76-(13)(3.05) \\ \frac{3.05^2}{2}a=-47.41 \\ 3.05^2a=-94.82 \\ a=-\frac{94.82}{3.05^2} \\ a=-10.19 \end{gathered}[/tex]

Therefore, the acceleration is -10.19 cm/s²

A guitar player tunes her strings so thatthere is a beat frequency of 1.0 Hzbetween them. If one string has afrequency of 220 Hz, what is thefrequency of the other string? (Thereare two possible answers; give one.) -(Unit = Hz)

Answers

Fb = beat frequency = 1 Hz

F1 = frequency 1 = 220 Hz

Fb = l F1 - F2 l or

Fb= l F2 - F1 l

Replacing:

1 = 220 - F2

f2 = 220 - 1

f2 = 219 HZ

what happens to F 1. m is doubled2. m is tripled3. m an m are both doubled4. m is halved5. r is doubled6. r is tripled7. r is increased 10 times8. r is halved

Answers

The expression for the gravitational force is given as:

[tex]F_g=\frac{GMm}{r^2}[/tex]

1. When M is doubled, then force will also be doubled as Force is directly proportional to the mass.

[tex]\begin{gathered} F\alpha M \\ F\alpha m \\ F\alpha\frac{1}{r^2} \end{gathered}[/tex]

2. When the m is tripled, then force will be tripled.

[tex]\begin{gathered} F^{\prime}\alpha\frac{M\times3m}{r^2} \\ F^{\prime}\alpha\frac{3Mm}{r^2} \\ F^{\prime}\alpha3F \end{gathered}[/tex]

3. When both M and m are doubled then, the force will become four times.

[tex]\begin{gathered} F^{\prime}\alpha\frac{2M\times2m}{r^2} \\ F^{\prime}\alpha4F \end{gathered}[/tex]

4.When m is halved, then Force will be halved, as force is directly proportional to the mass.

[tex]\begin{gathered} F^{\prime}\alpha\frac{M\times\frac{m}{3}}{r^2} \\ F^{\prime}\alpha\frac{Mm}{3r^2} \\ F^{\prime}\alpha\frac{F}{3} \end{gathered}[/tex]

5. When distance is doubled, then force will become one-fouth.

[tex]\begin{gathered} F^{\prime}\alpha\frac{Mm}{(2r)^2} \\ F^{\prime}\alpha\frac{Mm}{r^2}\times\frac{1}{4} \\ F^{\prime}\alpha\frac{F}{4} \end{gathered}[/tex]

6.When the distance is tripled, the force will become one-ninth.

[tex]\begin{gathered} F^{\prime}\alpha\frac{Mm}{(3r)^2} \\ F^{\prime}\alpha\frac{Mm}{r^2}\times\frac{1}{9} \\ F^{\prime}\alpha\frac{F}{9} \end{gathered}[/tex]

7. When the distance is increased to ten times, the force will become one-hundredth.

[tex]\begin{gathered} F^{\prime}\alpha\frac{Mm}{(10r)^2} \\ F^{\prime}\alpha\frac{Mm}{r^2}\times\frac{1}{100} \\ F^{\prime}\alpha\frac{F}{100} \\ 8.\text{ When the distance in halved, the force will be four time.} \\ F^{\prime}\alpha\frac{Mm}{(\frac{r}{2})^2} \\ F^{\prime}\alpha4\frac{Mm}{r^2} \\ F^{\prime}\alpha4F \end{gathered}[/tex]
Other Questions
What is the Area of a circle with the radius of 5 3. Make a claim as to why the leading cause of residential fires is not the leading cause of residential fire deaths.4. What percentage of fires are intentionally set? Why might someone intentionally set a fire?5. Do you think there would be more structural fires in the summer or winter? Explain your answer. find the error & explain why it is wrong: jon had to solve the equation below. what did he do wrong? Complete the table for each function. Then answer the questions that follow.X Y=4x Y=4x^2 Y=4^x0 0 0 11 4 a b2 c 16 d3 e f ga=? b=?c=?d=?e=?f=?g=? For the data in the table, does y very direction with x? If it does, write an equation for direct variation. Show your work. (2 points) Hint: y=kx is the formula for direct variation. Find k, then substitute it back in to the formula. The sum of two numbers is 16. Their diference is 20. Find the numbers i need the range for city A and city B, and the standard deviation for city A and city B true or flase? Any method (elimination, substitution, or graphing) can be used to any possible system of linear equations. Suppose a Ballon rises 250 ft during the first minute................. The rumble feature on a video game controller is driven by a device that turns electrical energy into mechanical energy. This device is best referred to as _________?A. an electric generatorB. an electromagnetC. a solenoidD. a motor 15. Write the equation of a horizontal line going through the point (5,4). Equation: using the pythagorean theorem and finding distances What is the mean of the data?Number of Letters inOur First Name3 XX4 XX5 XXXX6 X7 X8 X I keep getting 3 but it was marked wrong with the correct answer of 5. Where am I going wrong? 10 points and brainliest to first correct answer! please answer both! The perimeter of the quadrilateral ABCD is 105 inches what is the Lenght of segment BC Consider the polynomial function p given by p(x) = 728 2x2 + 32 + 10. Evaluate the functionat x = -3 Which fate did the Matamoros Expedition have? please helpQuestion 8 options:A Texan army commanded by Francis Johnson captured Matamoros.Mexican soldiers burned the city, making the expedition pointless.Texans cancelled the expedition because a large Mexican army was marching north.Mexican troops defeated the Texan force attempting to capture the port. describe your so how to find the following product using both an algorithm in the diagram 3 * 3/4 Where must you store potentiallyhazardous foods?O a. Dry storageO b. Break roomO c. Cold storage Describe the end behavior for the polynomial function described below.